Docsity
Docsity

Prepare for your exams
Prepare for your exams

Study with the several resources on Docsity


Earn points to download
Earn points to download

Earn points by helping other students or get them with a premium plan


Guidelines and tips
Guidelines and tips

BARKLY FNP REVIEW EXAMS TEST BANK WITH ACTUAL CORRECT QUESTIONS AND VERIFIED DETAILED R, Exams of Nursing

BARKLY FNP REVIEW EXAMS TEST BANK WITH ACTUAL CORRECT QUESTIONS AND VERIFIED DETAILED RATIONALES ANSWERS 2024 LATEST ALREADY GRADED A+ BARKLY FNP REVIEW EXAMS TEST BANK WITH ACTUAL CORRECT QUESTIONS AND VERIFIED DETAILED RATIONALES ANSWERS 2024 LATEST ALREADY GRADED A+ BARKLY FNP REVIEW EXAMS TEST BANK WITH ACTUAL CORRECT QUESTIONS AND VERIFIED DETAILED RATIONALES ANSWERS 2024 LATEST ALREADY GRADED A+ BARKLY FNP REVIEW EXAMS TEST BANK WITH ACTUAL CORRECT QUESTIONS AND VERIFIED DETAILED RATIONALES ANSWERS 2024 LATEST ALREADY GRADED A+ BARKLY FNP REVIEW EXAMS TEST BANK WITH ACTUAL CORRECT QUESTIONS AND VERIFIED DETAILED RATIONALES ANSWERS 2024 LATEST ALREADY GRADED A+ BARKLY FNP REVIEW EXAMS TEST BANK WITH ACTUAL CORRECT QUESTIONS AND VERIFIED DETAILED RATIONALES ANSWERS 2024 LATEST ALREADY GRADED A+ BARKLY FNP REVIEW EXAMS TEST BANK WITH ACTUAL CORRECT QUESTIONS AND VERIFIED DETAILED RATIONALES ANSWERS 2024 LATEST ALREADY GRADED A+ BARKLY FNP REVIEW EXAMS TES

Typology: Exams

2023/2024

Available from 03/22/2024

chokozilowreh
chokozilowreh 🇺🇸

3.6

(13)

883 documents

1 / 269

Toggle sidebar

Related documents


Partial preview of the text

Download BARKLY FNP REVIEW EXAMS TEST BANK WITH ACTUAL CORRECT QUESTIONS AND VERIFIED DETAILED R and more Exams Nursing in PDF only on Docsity! 1 | P a g e BARKLY FNP REVIEW EXAMS TEST BANK WITH ACTUAL CORRECT QUESTIONS AND VERIFIED DETAILED RATIONALES ANSWERS 2024 LATEST ALREADY GRADED A+ A 4-year-Old girl complains of localized, painful swelling on her left eyelid. Her parents say that an abcess on her eyelid seems to be causing the swelling and redness. Which of the following would be the most likely cause? Hordeloum Which cranial nerves does not control eye movements CN II Of the following, which condition is most likely to develop as a result of pressure on the cornea in cases of chalazia? astigmatism Which of the following pathogens is least likely to cause acute sinusitis in the adult-gerontology patient? Staphylococcus aureus The American Academy of Ophthalmology recommends that all patients recieve a complete screening for glaucoma by at least what age? By age 40 Shawn, a 23 year old male, has been experiencing nasal muscosal discharge, headache, cough, and malaise for approximately 5 days. You suspect that he has a cold. Which findings would best suggest an alternative diagnosis? He has occasional muscle aches What are hyperopia and myopia also known as, respectively? Farsightedness and nearsightedness Your patient lives in Palm Desert and presents to your practice with this finding. He describes it as painless, though sometimes it "feels like there's something sticking to my eye." What is the diagnosis? Pterygium 2 | P a g e Which of the following findings is present in 90% of all cases of cholesteatoma? Tympanic membrane perforation When performing an eye exam, if you have difficulty visualizing the macula, what instruction should you give the patient? Look into the light Which of the following does NOT accurately describe the usual mucosal findings of a patient with a typical case of sinusitis? a. Dry b. Reddened c. Foul-smelling d. Discolored Dry A patient presents with hearing loss and painless otorrhea. Upon physical exam, you note the canal is filled with mucopus and granulation tissue. Tympanic perforation and ossicular damage are also noted. What is the best way to treat this patient's condition? Refer the patient for surgery A patient presents with complaints of a burning sensation in the left eye. A physical examination reveals redness and watery, nonpurulent discharge. The patient presents with no other physical symptoms or sensations. Based on the patient's signs and symptoms, which type of conjunctivitis is the most likely diagnosis? Viral conjunctivitis A male patient in his early 20s who was diagnosed with a stye in his left eye has been applying a warm compress to the affected area for more than a week as part of a home remedy. Despite this treatment, the patient's stye is still present. What further treatment would be best to recommend at this time? Prescribe erythromycin topical ointment 5 | P a g e A patient presents with a white. pimple-like small lesion protruding on his eyelid. It is tender, firm, and discrete. Which of the following should you most likely chart? Hordeolum A healthy, fit, 41 year old male says the the has been experiencing several problems that manifested about 2 days ago after his head was slammed onto the mat while wrestling at a mixed martial arts school. His initial complaints were dizziness and ringing in the ears. Since then, he has been experiencing short periods of dizziness and disorientation, ringing in the ears, and blurred vision. An ear exam shows no signs of inflammation or exudate. The patient may require a further diagnostic assessment for all of the following conditions except: a. Retinal detachment b. Vertigo c. Otitis externa d. Hearing loss Otitis externa Nonviral otitis media is most commonly caused by what pathogenic bacteria? Streptococcus pneumoniae Your patient complains that his eyes "just aren't the same color anymore." What is the most likely diagnosis, given the picture? Arcus senilis During an eye exam, a patient's optic cup-to-disc ratio indicates that the size of the optic cup is larger than one half of the diameter of the optic disc. Which optic disease does this finding most strongly indicate? 6 | P a g e Glaucoma An elderly male presents with symptoms that include a burning sensation in the eye and red, scaly flakes around the eyelids, which are covered with thick crusts. Based on the patient's presentation, which of the following medications should be prescribed to treat the patient's condition? Bacitracin or erythromycin A patient comes to your practice with symptoms of fever and pain in the throat. Suspecting strep pharyngitis, you assess the patient under the guidelines of the Center criteria. Under these standards, which of these findings would not be confirmatory of strep pharyngitis? Presence of persistent cough Joe, a 54 year old male, comes to your practice with complaints of frequent drainage in his ear and decreased hearing for about 2 years. You examine his ear with an otoscope, and discover that his tympanic membrane is perforated. In addition, his ear canal is filled with mucous and peeling layers of scaly epithelium. Which conditions does the patient most likely have? Cholesteatoma Cataracts are most commonly associated with which condition? Diabetes You know that the following factors may usually contribute to sensorineural hearing loss except: a. Close range to an explosion b. Acoustic neuroma c. Perforated tympanic membrane d. Long-term exposure to loud music Perforated tympanic membrane If a patient with a sore throat has been advised to avoid contact sports, his condition most likely stems from which of the following viruses? Epstein-Barr virus 7 | P a g e During an eye exam, a 65 year old man complains of blurred vision and flashes of light in his periphery, which began to manifest within the past day. The patient was diagnosed with type 2 diabetes mellitus at age 55. Given these findings, which of these is the most likely eye condition, and how would the nurse practitioner best manage this condition? Retinal detachment and should be referred to a surgeon All of the following treatment options are commonly used to manage vertigo meclizine (Antivert) promethazine (Phenergan) dimenhydrinate (Dramamine) transdermal scopolamine The practitioner knows that which agent is the least effective in the treatment of candidiasis balanitis? Griseofulvin Clotrimazole Steroids Miconazole griseofulvin Which skin skin cancer has the highest mortality rate? Malignant Melanoma A 27-year old female presents with a ring-shaped skin lesion with a central clearing on her left leg. The skin lesion would most accurately be charted as? Annular A 19-year-old female presents with the following rash and describes the itchy sensation as intense. Additionally, she says that she has been experiencing occasional itchy flare up since childhood, yet her symptoms last no more than a few days. Which condition do her complaint most likely indicate? Eczema 10 | P a g e Cryotherapy Steroid injection Steroid injection A patient presents to the clinic with the following lesion. Based on the presentation, which is the most likely diagnosis? Anthrax Which of the following treatments is a priority when treating for frostbite? treat for pain and assess for hypothermia. than soak in water 100 degrees for 15 minutes Ant bites, elevated nevi, and common skin warts are examples of which type of skin lesion? papule Ephelis freckle Sharon, age 45, comes to your office with asymmetrical skin lesions that are elevated and are larger than 6 millimeters. The color and lesions varies from brown to purple, and the borders are irregular. Which of the following is the most likely diagnosis? Malignant Melanoma A 22 year old male comes into clinic complaining of pain in the goin area, Upon examination of his groin area, you identify abscess formations. Based on the patient's presentation, which is the best diagnosis for his skin condition? Hidradenitis suppurativa 11 | P a g e An otherwise healthy, 28-year-old male presents with fever, chills, and joint pain. The patient says he went on a hiking trip in a wooded area two weeks prior. Upon examination, you notice a circular pattern resembling a bull's eye on his back. Which test would you initially order to narrow the differential diagnosis of the patient's most likely condition? ELISA Which of these would not be considered a first line treatment for inflammation of the glans penis caused by candida albicans? Miconazole Ketoconazole Clotrimazole Fluconazole Ketoconazole A young adult presents with the following lesion on his face. The rash is most characteristic of which condition? Impetigo Which of the following best describes the appearance of Auspitz sign when psoriasis scales are removed? droplets of blood Which acne treatment would NOT be recommended to a patient who is going to be on a 3-month backpacking trip in the desert and will not have access to electricity? Benzoyl peroxide 12 | P a g e Clindamycin Erythomycin-benzoly peroxide Clindamycin-benzolyl peroxide gel Erythomycin-benzoly peroxide A fair skin 47-year-old undergoing a periodic health assessment presents with pink patches on her upper chest. She says that the patches appeared about a year ago. Within the past couple of weeks, though, the patches have started to itch. Which would best treat the patient's most likely condition? liquid nitrogen Which of the following would a nurse practitioner be likely to prescribe for a minor bacterial skin infection? Topical antimicrobial. A 25-year old presents with with the following lesions. which of these would be the most effective to treat the patients condition? miconazole clindamycin betamethasone Doxycycline betamethasone If a patient is experiencing a painful, blistering eruption in a dermatolmal distribution that resembles the following, which is the most likely diagnosis ? herpes zoster Which of the following is NOT a Dermatophyte infection? Candida balanitis Tinea capitus Ringworm Onychomycosis candida balanitis 15 | P a g e Metabolic Syndrome (Syndrome X) A genetic metabolic disorder characterized by diabetes, hypertension, atherosclerosis, centrally distributed obesity, and elevated blood lipids Components of metabolic syndrome Waist circumference > 40 inches men (102 cm) >35 inches women (89cm) Blood pressure > 135/85 Fasting blood glucose > 100 High-density lipoprotein < 50 women < 40 men Triglyceride level >150 Which of the following is NOT a common component of the pathology for type 1 DM? a. Human leukocyte antigens b. Islet cell antibodies c. Peripheral insulin resistance d. Ketone development Peripheral insulin resistance Type 2 diabetes mellitus is associated with insulin resistance. Which of the following statements about insulin resistance is true? Insulin resistance may improve with weight loss The metabolic syndrome, also know as syndrome X, is associated with the risk of DM 2. It is characterized by a group of predisposing factors that include obesity and hypertension. What additional risk factors are associated with syndrome X? High triglyceride levels and low HDL levels What is the preferred diagnostic test for visualizing Grave's ophthalmopathy? Magnetic resonance imaging 16 | P a g e Which of the following is most likely to be found on the fundoscopic examination in a patient with untreated POAG? excessive cupping of the optic disk A 35-year-old woman is complaining of gradual weight gain, lack of energy, and amenorrhea. The urine pregnancy test is negative. The CBC shows a hemoglobin of 13.5 g/dL and an MCV of 84. The NP suspects that the patient may have hypothyroidism. The TSH is 10mU/L. Which of the following is the next step in the evaluation? Select one: a. Check the thyroid panel b. Check the total T3 and T4 levels c. Check for Antithyroid peroxidase antibodies d. Recheck the TSH in 4-6 months Check the thyroid panel Patients diagnosed with hyperthyroidism face a variety of treatment options, depending on etiology, severity, and other factors. Radioactive iodine therapy is one such option that should be specifically considered in which of the following patients A female diagnosed with Graves' disease A patient is at risk for secondary hypothyroidism if they have which condition? Pituitary adenoma Epidemiologic studies show that Hashimoto's disease occurs most commonly in: Middle-age to older women Susan, age 25, presents to the clinic with an altered level of consciousness. Kussmaul's breathing, polyuria, and fruity breath. Upon looking at her medical file you note she has type 1 diabetes. You immediately direct the patient to an emergency room for treatment and further evaluation. Which lab results would you LEAST expect Susan to have? Select one: a. Hyperglycemia b. Ketonuria c. Alkalosis d. Hyperkalemia Alkalosis (Acidosis) 17 | P a g e Which of the following methods would be most helpful in counteracting the somogyi effect in a diabetic patient Decrease the patient's at bedtime dose of insulin Which of the following conditions is the most common cause of sudden death among athletes? Hypertrophic cadiomyopathy The most common comorbidities that occur with type 2 diabetes mellitus are: Hypertension, hyperlipidemia, obesity A 36-year-old patient presents to your office with concerns over experiencing uncontrollable fine tremors in her hands. When you ask her about other concerns, she tells you she has been feeling anxious and nervous, and that her emotions seem to have been fluctuating "up and down" over the past 2 weeks. Her complaints are most likely associated with which condition? Graves' disease A nurse practitioner is slowly and cautiously rewarming a patient with blankets to avoid circulatory collapse. Which condition is the nurse practitioner most likely trying to prevent? Myxedema coma Your patient, Franklin, age 64, has the following lab values: elevated TSH, low T4, and decreased T3. Which of the following drugs should be prescribed to treat his condition? Levothyroxine Kussmaul's breathing is a finding most commonly associated with which condition? Diabetic ketoacidosis What complication usually occurs as a result of type 2 DM, causing patients to produce insufficient amounts of insulin to prevent severe hyperglycemia, osmotic diuresis, and extracellular fluid depletion Hyperosmolar hyperglycemic non ketosis Most cases of epistaxis occur at Kiesselbach's plexus (Little's area) Key diagnostic findings in Primary open-angle glaucoma (POAG) include which of the following? intraocular pressure greater than 25 mm Hg. 20 | P a g e Hyperosmolar hyperglycemic non-ketosis A 45-year-old patient with nausea and a fever of 104.4 F. He said the fever appeared earlier in the day, and that he has vomited more than once prior to arriving at your office. During the examination, he states that he feels dizzy and lightheaded whenever he stands up. His vitals indicate that he has low blood pressure. Based on these findings, which of these is the most likely diagnosis? Addison's disease Which of these statements is true regarding hyperthyroidism? The incidence of hyperthyroidism is higher in women than in men Which condition involves damage to the adrenal glands includes metastatic cancer as a possible etiology? Addison's disease Which of the following processes is most commonly associated with type 2 diabetes? Secretory defect causing a resistance to insulin Appropriate thyroid hormone biosynthesis is dependent on the dietary intake of: Iodine When caring for a patient with DM, HTN, and persistent proteinuria, the NP prioritizes the choice of antihypertension and prescribes: Fosinopril A 50-year-old woman of Irish descent presents with history of lethargy, feeling weak, nausea, anorexia with diarrhea and abdominal pain. The woman's' skin appears tanned with hyperpigmentation of the nipple area, the gums and the lips. The electrolyte panel reveals hyperkalemia and hyponatremia. She reports craving salty foods. Which of the following is most likely? Addison's disease 21 | P a g e Jonathan, a 49-year-old-patient complains of frequent dizzy spells. His lab tests yield an elevated erythrocyte sedimentation rate, normal iodine uptake, normal thyroid stimulating hormone levels and low plasma cortisol. In addition, you note the appearance of hyperpigmentation on the patient's skin, particularily in skin creases. What would be the most likely diagnosis? Addison's disease The nurse practitioner who suspects that one of her hypertensive patients has Cushing's syndrome would expect to find which of the following laboratory results? a. Hyponatremia b. Hypoglycemia c. Elevated serum cortisol levels d. Decreased urine 17-ketosteroids Elevated serum cortisol levels Which of the following is not an expected cause of Cushing's syndrome? a. Chronic adminstration of glucocorticoids b. Adrenocorticotropic hormone hypersecretion c. Autoimmune destruction of the adrenal gland d. Adrenal tumors Autoimmune destruction of the adrenal gland You are seeing a patient who was diagnosed with hyperthyroidism and had treatment for thyroid storm. You advise the patient that which of the following should be avoided to decrease the risk of developing another thyroid crisis? Aspirin When starting an elderly patient on a new prescription of levothyroxine (Synthroid), the NP should keep in mind that the rationale for starting an elderly patient on a lower dose is which of the following? Due to its cardiac effects A middle-aged female who is complaining of gradual weight gain, lack of energy, dry hair, and an irregular period over an 8-month period. The routine annual laboratory testing showed a TSH result of 10 mU/L. The NP decides to order a thyroid panel. The TSH is 8.5 mU/L and the serum T4 is decreased. The patients BMI is 28. The heart and lungs exams are normal. Which of the following is the best treatment plan? Start the patient on levothyroxine (Synthroid 0.25 mcg daily) Which of the following is most true regarding the treatment of hypothyroidism with levothyroxine? 22 | P a g e The dosage should be given every day What would be your most likely reason for prescribing Lugol's solution to a patient? Reduce vascularity of the gland A 36-year-old patient presents with concerns over experiencing uncontrollable fine tremors in her hands, feeling anxious and nervous, and that her emotions seem to have been fluctuating "up and down" over the past 2 weeks. Her complaints are most likely associated with which conditions? Graves disease A patient diagnosed with hyperthyroidism is about to undergo thyroid surgery. Which of the following factors would be most reliable as an indicator of a euthyroid state? Thyroid stimulating hormone is normal The best screening test for detecting and monitoring both hyperthyroidism and hypothyroidism is: Thyroid stimulating hormone The most sensitive laboratory indicator of overall thyroid function is to evaluate the level of circulating: TSH With regards to pathology of diabetes, which of the following factors plays a principal role in causing type I DM? Toxic environmental insult to pancreatic beta cells A female presents to the urgent care unit with complaints of constant nausea, weakness, and fatigue. She reports that she has been feeling extremely thirsty since her symptoms began 2 days ago, and she has been urinating a lot, which she attributes to constantly drinking water. The physical exam reveals the patient has a normal blood pressure and heart rate. A urine test indicates ketones. For which condition should the patient be further evaluated? Type 1 diabetes Which of the following groups has been recommended to be screened for thyroid disease Women 50 years or older Which of the following would most likely result from a breakdown in the body's ability to either produce or utilize insulin in type 1 diabetic patients? Inappropriate hyperglycemia You are treating a 55-year old male who is at risk of developing diabetes. When making recommendations about diet, what should you tell the patient about carbohydrate intake? Advise to take 55-60% of carbohydrate caloric intake Which of the following characteristics applies to type 2 Diabetes? 25 | P a g e Among patients experiencing meniscus knee injuries, which type of tear is typically the most common ? Medial meniscus. Think pop and lock with grapefruit swelling Tyler, age 20, fell during a basketball game and injured his left quadricep. In addition to recommending rest and immobilization, you want to help him with the pain. Which of these medications is least likely to see use in the initial treatment of Tyler's injuries? A. Naproxen B. Celecoxib C. Metaxalone D. Tramadol Celecoxib The findings of osteophytes, junta-articular sclerosis, and subchondral bone on an x-ray would most strongly indicate that a patient has osteoarthritis. Positive radiographic findings are most instrumental in diagnosing which of these conditions? A. Plantar fasciitis B. Bursitis C. Carpal tunnel syndrome D. Osteoarthritis Osteoarthritis Morton's neuroma is a benign neuroma that causes a compression neuropathy on an intermetatarsal nerve, most commonly on which two intermetatarsal spaces? Third or fourth inter metatarsal spaces A woman comes to your practice with complaints of stiffness in her shoulder and "this terrible ache" in her hips. You also find that she has anemia and mild fever. Which of the following is the patient most likely experiencing? Polymyalgia rheumatica 26 | P a g e A hockey player presents with pain around the sternum that worsens whenever he takes a deep breath. He says his symptoms began nearly a week ago after receiving a hard blow to the chest during a game, and that the pain has been getting progressively worse. for which condition should the patient most likely be first evaluated? Costocondritis A 71-year-old male with an active lifestyle presents to your practice with complaints of pain in his right shoulder. Which of these statement from the patient would most strongly indicate bursitis? "It hurts when I move my arms over my head." Which of the following areas is not a common site of bursitis? A. Postpatellar (prepatellar) B. Olecranon (knee) C. Subdeltoid (shoulder) D. Ischial Postpatellar One of the most common sites for bursitis is the prepatellar, not post patellar, area. Other common sites for bursitis include the olecranon, subdeltoid, and ischial areas. Which of these tests would provide the best assessment for carpal tunnel syndrome? Phalen's test "Locking" of the knee usually indicates loose bodies in the affected area. What other type of knee injury would this presentation most likely suggest? A. Ligament tear B. Knee sprain 27 | P a g e C. Soft tissue injury D. Meniscal tear Meniscal tear Which of the following musculoskeletal disorders would most likely compel a referral for cryogenic neuroablation? Morton's neuroma A 42-year-old secretary is diagnosed with carpal tunnel syndrome, and you are advising her on proper forms of management. Which of the following medications would be best for early treatment of her symptoms? Celecoxib , and other NSAIDs may see use as the first step in pain management for carpal tunnel syndrome. What is the name of the occupational splint used for carpal tunnel syndrome cock up splint For swollen, painful knees, what is the easiest most sensitive test to assess for anterior and posterior cruciate ligament tears? Lachman's test What test usually assesses for medial or lateral collateral ligament damage or meniscus injury? Apley's grind test 30 | P a g e A female presents with an ankle sprain. She is unable to walk unassisted, an egg-shaped swelling is identified around the affected area. The patient's ankle sprain should be classified as which of the following? Grade 3 A patient comes to your practice with a knee that keeps "locking," making it difficult for him to walk. You perform the apple's grind test on the patient to determine the presence of soft tissue damage. What would be the best method for carrying out this test? Flexing the knee 90 degrees, and then applying pressure to the heel while rotating the lower leg. (pain or click is positive) A 26-year-old male was recently diagnosed with plantar fasciitis. He says he experiences sharp pain and stiffness in the affected foot each morning; however this pain usually fades by the time he's ready for work. Which treatments would have the lowest priority at this time? A. Orthotics B. Physical therapy C. Night splints D. Opioids Opioids An Adolescent has experienced a rupture in the growth plate at the tibial tuberosity as a result of stress on the resulting condition. Which of these statements would you most likely make? "The pain will often be worsened by running, jumping, and climbing stairs." A 20-year-old patient is brought to the clinic after crushing his finger in a door. The blow has caused throbbing in the finger, blue and black discoloration of he nail, and bleeding from he nail bed. Which methods of management would be least appropriate at this time? A. Hydrocodone and acetaminophen B. Naproxen C. Trimcinolone hexacetonide D. Ibuprofen Trimcinolone hexacetonide After identifying the possibility of Osgood-Schlatter disease during a physical exam, what test would best confirm the diagnosis X-ray 31 | P a g e When performing an x-ray on a patient with osteoarthritis which of the following findings would be expected? A. Osteophytes B. Narrowing of joint space C. Subchondral bone D. Juxta-articular sclerosis When performing an x-ray on a patient with rheumatoid arthritis which findings would expected? A. Joint swelling B. Osteopenia C. Progressive cortical thinning D. Joint space narrowing Your male patient says that he experiences severe, sharp pain in the bottom of his right foot when he wakes up in the morning, which subsides throughout the day. He describes a dull pain in the heel and a throbbing ache in the arch of his foot. Plantar fasciitis Which antimalarial agent is used in the treatment of Rheumatoid arthritis? hydroxychloroquine Of the following, when is an antinuclear antibody examination of a knee injury most often indicated? If rheumatoid arthritis is suspected. What method of management is recommended for plantar fasciitis? NSAIDS Corticosteroids orthotics Night splints Physical therapy The following methods of management are recommended for Morton's neuroma? 32 | P a g e Orthotics and Corticosteroids Referral for cryogenic neuroablation or neurectomy STDs matched with pathogens PID- Neisseria Gonorrhoeae & Chlamydia Lymophogranuloma venereum- chlamydia trachomatis Chancroid- Haemophilus ducreyi Condyloma acuminata (genital warts)-human papillomavirus (HPV). Syphilis- Treponema Pallidum Luis, a 42-year old, presents with a painless indurated ulcer on his anus. What stage of syphilis are you most likely in? primary stage: What two antibiotics can be given to a patient with a penicillin allergy that has syphilis? doxy or erythromycin What is the male to female transmission rate of gonorrhea on initial exposure? 80-90% A 23 year old with pearly white papules around anogenital region. Aprox 2 mm wide, round, smooth and firm. No other signs or symptoms? What do they have? Molluscum Contagiosum Which finding from a patient's cervical cytology would best confirm condyloma acuminata? squamous intraepithelial lesions What STD is caused by a parasitic pathogen? Chlamydia trachomatis is a parasitic intracellular obligate Which statement most accurately reflects the relationship between HIV and CD4 cells? HIV uses CD4 cells as receptors and reservoirs Pt has been hospitalized for meningitis, what STD would you most likely suspect the patient to have? 35 | P a g e Molluscum contagiosum and cryoanesthesia with liquid nitrogen is the best form of removal. A 30 year old with yellow-green penile discharge, testicular pain, and nausea and vomiting. What medication should be included for uncomplicated gonorrhea ceftriaxone 500mg IM plus doxy Which vaccine helps prevent and protect against hep B? Energix- Recombivax-B, Heplisav-B. Twinrix is a combination of Hep-A and Hep-B what keratolytic agents are used to treat condyloma acuminata? podofilox imiquimod sinecatechins What is the 3rd most commonly reported STD in the US? Syphilis Chlamydia is most common, 2nd=gonnorhea Swollen lympnodes in the inguinal region is common in what conditions? LVG, chancroid, syphilis A patient with advanced case of LGV reveal buboes in the vulvovaginal area. Which treatment is recommended to prevent ulceration? Needle aspiration A patient who presents with white purulent penile discharge and six maculopapular pustular lesions. He also complains of pain in his testicles and frequent nausea. what is his most likely diagnosis? Gonorrhea, Lesions are not always present but when they are they are painful and there can be as many as 5-40. what is the typical window of time in which an infected person most commonly converts from HIV neg to positive? Seroconversion, takes between 3 weeks and 6 months after initial infection A 44 year old male sent to the hospital for an STD that is characterized by hemiparesis, hemiplegia, cardiac insufficency. Given the most likely stage of tertiary syphilis. Which of the following signs and symptoms would you likely expect? Aortic aneurysm, leukoplakia, meningitis. 36 | P a g e What are common signs and symptoms of an initial outbreak of herpes? dysuria, fever, malaise The following are common known causes of primary amenorrhea except for: a. Turner syndrome b. Vaginal agenesis c.Resistant follicular stimulation d. Endocrine imbalance Resistant follicular stimulation Of the following, who is most likely to experience symptoms of premenstrual syndrome/premenstrual dysphoric disorder? A fit, healthy 33-year-old with a high stress job You order a referral to further evaluate your 19-year-old patient who reported symptoms of cramping, pain in her upper thighs, nausea, and fatigue during the first few days of her menstrual cycle. For which do you make a referral? Primary dysmenorrhea A 64-year-old male says that he has trouble "getting started" when he wishes to void, and the stream "keeps leaking" when he tries to terminate it. He also wakes frequently with the urge to urinate.He mentions with some embarrassment that he "let go" while celebrateing a victoy for his sports team. Which diagnostic test should you order initially? a. Prostate-specific antigen b. Gram stain c. Urinalysis d. 24-hour urine collection Urinalysis Which has the fastest onset and longest duration for erectile dysfunction? Tadalafil or Avanafil works in 15min. Take with or without food The results of a patient's cervical cytology test indicate "atypical squamous cell of undetermined significance". Which test would you least likely order as a followup? a. human papillomavirus testing b. a second pap smear c. ultrasound d. colposcopy 37 | P a g e c. ultrasound Which patient has the most substantial signs for developing osteoporosis? a 5'4" asian female weighing 90 lb d. According to the most recent CDC recommendations for treating PID, which of the following drugs would not be paired with doxycycline? a. cefoxitin b. probenecid c. metronidazole d. butoconazole butoconazole CDC recommendations for treating PID ceftriaxone 500 mg IM in a single dose6 PLUS doxycycline 100 mg orally 2x/day for 14 days WITH metronidazole 500 mg orally 2x/day for 14 days OR cefoxitin 2 gm IM in a single dose AND probenecid 1 gm orally, administered concurrently in a single dose PLUS doxycycline 100 mg orally 2x/day for 14 days WITH metronidazole 500 mg orally 2x/day for 14 days You are seeing a 24 year old female who states that she has been experiencing severe cramping pain during her last 2 menstrual cycles. Which of the following conditions is the patient experiencing? secondary dysmenorrhea Your post-menopausal patient complains of "hot flashes all the time" and insists that her quality of life is poor. You consider hormone therapy with which fact in mind? Breast cancer has been linked to those taking HT A patient diagnosed with abnormal uterine bleeding is undergoing further evaluation to determine the etiology. Which condition is least likely to be the cause of the patient's abnormal uterine bleeding? a. Polycystic ovarian disease b. Primary dysmenorrhea c. Perimenopause d. Immature hypothalamic-pituary-ovarian axis Primary dysmenorrhea A 79-year-old male having trouble with my urine. His prostrate-specific antigen was noted to be 6.0 ng/ml. The nurse practitioner considers a referral for which of the following tests at this time? Transrectal ultrasound A 19 year old patient tells you that she normally experiences pain during the first few days of the menstrual cycle. She describes the pain as a cramp that radiates to her back and upper thighs, usually 40 | P a g e Which pathogen would be the most likely cause of prostatitis in a 64-year old male ? Escherichia coli Which of the following is the least definitive test in diagnosing PID? a. sexually transmitted disease screening b. Erythrocyte sedimentation rate test c. Ultrasound d. TSH test d. TSH test Which of the following is the most accurate statement regarding the biological effects during menopause? The vagina experiences decreased stimulation as the epithelium atrophies Which following medications you will prescribe for Chlamydia trachomatis caused PID? Ceftriaxone and doxycycline with metronidazole A patient complaining os intense itching in her vaginal area and yellowish-green discharge with smells like old gabage. Inspecting the vulvovaginal region, you find erythema and red spots around the vagina. You order a wet prep test and expect which of the following results? Motile trichomonads Whch of the following drugs is not a phosphodiesterase inhibitor? a. Sildenafil b. vardebafil c. Chlorophyllin d. Tadalafil Chlorophyllin All the following tests would be used to diagnose pelvic inflammatory disease except : a. Venereal Disease Research Laboratory test b. ESR test c. C-reactive protein test d. ultrasound Venereal Disease Research Laboratory test Iron deficiency anemia results in a decrease in the amount of iron available for which process? Red blood cell formation A 45 year old patient presents to the clinic complaining of weak arms, pins and needles, and fingers that can't keep a pen steady. A blood panel shows that his hemoglobin, hematocrit and red blood cell count are decreased, whereas his mean corpuscular volume is increased. Which follow-up diagnostic would be least helpful in confirming the most likely condition? 41 | P a g e A. Citrate agar gel electrophoresis B. Anti-intrinsic factor test C. Schilling test D. Antiparietal cell antibody test. Citrate agar gel electrophoresis The TNM classification for a patient reads as follows: T2, N1, M0. All of the following inferences are supported by the patient's TNM classification except: A. The patient's bladder tumor has spread to the muscle of the bladder wall B. The patient's breast tumor has spread to axillary lymph nodes C. The patient's colon tumor has spread to nearby lymph nodes D. The patient's brain tumor has not spread to distant tissue The patient's brain tumor has not spread to distant tissue. Brain tumors do not have TNM classifications A patient comes to the clinic with complaints of weakness, fatigue, and difficulty breathing during mild exercise. His lab results indicate a low serum iron level, a low total iron binding capacity level, and a low hemoglobin level. However, his mean corpuscular hemoglobin count and mean corpuscular volume are both normal. What condition would the patient's symptoms and lab results most strongly indicate? Anemia of chronic disease Naomi, a 69 year old female presents to your clinic with complaints of fatigue, weakness, and breathlessness when working out. Her medical history reveals that she has renal failure. You order a blood test and the panel shows that her serum iron is low and her serum ferritin is elevated. Which of the following conditions is the most likely diagnosis? Anemia of chronic disease Identify the finding that best indictes pernicious anemia Presence of antiparietal cell antibodies A 47 year old male patient has signs of leukemia. Which of the following diagnostic tests would best distinguish acute leukemia from chronic leukemia? Peripheral blood smear Which of these anemias is properly paired with a dietary staple well-suited for treating the condition? Folic acid deficiency: peanut butter A patient's lab results show a mean corpuscular volume of 120 fL. Which of the following anemias would be most strongly indicated by this finding? A. Iron deficiency anemia B. Sickle cell anemia C. Folic acid deficiency D. Thalassemia Folic acid deficiency 42 | P a g e Non-Hodgkin's lymphoma is more likely than Hodgkin's disease to present with which characteristic? A. Unpredictable pattern of spread B. Reed-Sternberg cells (Hodgkin's) C. An unknown etiology D. Cervical adenopathy Unpredictable pattern of spread A 37 year old male presents with recurring headaches, fatigue, and a decreased desire to eat. An examination and lab panel reveals that the patient has a resting heartbeat of 122 bpm, yet a normal mean corpuscular hemoglobin concentration. Which condition is the patient most likely experiencing? Folic acid deficiency Sarah, a 35 year old female, presents to your practice with anorexia, glossitis, and dizzziness. She tests positive for both the Romberg and Babinski tests. Which of these is the most likely condition? Pernicious anemia (vitamin B12 anemia) Which of these values is within the normal range for mean corpuscular volume? A. 77 fL B. 92fL C. 101 fL D. 113 fL 92fL A patient presents to the clinic with complaints of a constant headache, fatigue, and a "racing heart." A blood panel reveals elevated mean corpustular volume, normal mean corpuscular hemoglobin concntration and a red blood cell folate of 83 mg/mL. Which of the following supplements would be best for the patient? Folate (less than 100) You suspect that your patient , Susana, has sickle cell a chronic, normocytic,hemolytic anemia. Select the best description of the diagnostic procedure that would best confirm the most likely condition? hemoglobin electrophoresis on citrate agar gel Benjamin, a 55 year old male, presents with complaints of fatigue, a heartbeat that "races" even when sitting down, and difficulty breathing when lifting heavy objects. What finding would most strongly indicate a vitamin B12 deficiency rather than a folic acid deficiency? Lack of intrinsic factor Joshua, a 45 year old male, comes to your practice with complaints of fatigue, weakness, and weight loss. After further investigation, you discover that he has generalized lymphadenopathy. You order a complete blood count, which reveals subnormal red blood cells and neutrophils. Given the most likely diagnosis of leukemia, you might recommend any of the following methods of management except: A. Symptomatic control B. Oral ferrous sulfate 45 | P a g e Most common leukemia in adults Chronic lymphocytic leukemia CLL Which of the following values best represents a normal red blood cell concentration in a 50 year old male patient? 48% (40-54%) A 35 year old male is presenting with complaints of fatigue and fever. No sign of infection is present Diagnostic tests show decreased red and white blood cells, as well as the presence of circulating blast cells. Which of the following is the most likely type of cancer based on these findings? Acute lymphoctic leukemia Your female sickle cell patient reveals to you that she plans to go on vacation in the next few months. Which of the following environments should you most strongly advise the patient against visiting? The mountains Your patient has just returned from competing in a soccer tournament in Denver. He complains of pain in his legs, nausea, tiredness, having trouble concentrating in school, and generally feeling unwell. The patient's history is positive for sickle cell trait. You order baseline studies to assess the patient. Which finding would you most expect to see in your patient? Acidosis Which of the following is the most approprate choice of management for moderate thalassemia? A. Red blood cell transfusion or splenectomy B. Chelation therapy C. Oral ferrous sulfate D. No treatment No treatment Which of the following factors most strongly contributes to the development of anemia of chronic disease? Decreased erythrocyte life span most common leukemia in adults chronic lymphocytic leukemia Which of the follow may likely result in lead poisoning? Indian herbal remedies, inner city playground near major highway, a red wagon from 1970 A mother replaced breast milk with whole cow's milk. Which dietary insufficiencies might the infant have insufficient iron 46 | P a g e A febrile 5-year old patient with sickle cell anemia has continued bedwetting. You order a urinalysis. Which of the following findings would you expect specific gravity of 1.008 (hypo concentrated) Most common pediatric leukemia Acute Lymphocytic leukemia Values of MCV=72, MCHC=30%, and reticulocyte count = 3%, respectively suggestive of thalassemia While going over lab results, you explain that chelation therapy is not recommended because the toddler's venous blood level concentrations do not reach which threshold 35 45 65 67 level 35 Susan, age 67, has been experiencing difficulty planning and organizing her weekly errands. She frequently has trouble articulating her thoughts and putting sentences together, and struggles with remembering how to cook her favorite meals. Which of the following medications would be least helpful in treating Susan? a. Tacrine hydrochloride b. Clonazepam c. Donepezil d. Galantamine Clonazepam Which of the following tasks is most pertinent in assessing a patient's attention in a mini-mental status exam? a. Repeating 3 objects b. Counting backwards from 100 in serial 7s c. Recalling 3 objects 5 minutes later d. Copying a design Counting backwards from 100 in serial 7s 47 | P a g e A patient complains of insomnia, indecisiveness, and fatigue. His physical examination indicates that the has lost a considerable amount of weight since his last visit 2 months prior. Which signs or symptoms would best support depression in his case? Anhedonia Margaret, a 29 year old, complains of feeling nervous, moody, and anxious. In addition, she experiences excessive tension and a "rapid heartbeat". To narrow your differential diagnosis, which of the following tests should be ordered first? TSH A 32 year old Asian female is in the emergency department after being assaulted by her husband. Her history shows that she has been seen 2 times within the past 6 weeks for bruises and facial abrasions. Which of the following statements best demonstrates the nurse practitioner's ethical responsibility to this patient before discharge? "I want you to know that this is not a normal relationship, and I would like for you to talk with someone here today about your family relationship." Which of the following drugs is an opiate antagonist used to help opiate-dependent patients with withdrawal? Buprenorphine (Buspirone) Which method is most commonly used in completed suicides? Firearms Although tricyclics and monoamine oxidase inhibitors are less efficacious for generalized anxiety, these medications are most likely to be useful in treating which issues associated with anxiety? a. Tachycardia b. Palpitations c. Panic attacks d. Breathlessness Panic attacks Possible cause of dementia Cortical atrophy Which of these would a practitioner use a test to assess a patient's cerebellar function? Balance and coordination You want to assess cerebral function in a patient experiencing memory loss. All of the following are main components of the examination that specifically assess cerebral function except: a. Thought processes b. Cognition 50 | P a g e b. Buspirone c. Imipramine d. Selegiline b. Buspirone Going by the standard CAGE mnemonic, which of the following questions is least helpful in the diagnosis for alcohol abuse? a. "Have you ever felt the need to cut down on your drinking? " b. "Have you ever justified having a drink for any reason? " c. "Have you ever felt guilt about your drinking? " d. "Have people annoyed you by criticizing your drinking?" "Have you ever justified having a drink for any reason?" Which of the following conditions accounts for the highest number of suicides? a. Alcoholism b. Drug Abuse c. Terminal diseases d. Mental disorders Mental disorders Agnosia the inability to recognize familiar objects. Of the following,which statement is most accurate regarding suicide? Most people who state intent to commit suicide actually follow through Rachel, age 33, comes to the clinic with complaints of frequent distraction, lack of focus, and disorientation. While in your office, Rachel's face goes blank and she forgets where she is, although she remembers a few minutes later. Which is the following is the most likely diagnosis? Delirium A patient complains of feeling anxious for no reason. He states he does not think he has panic attacks, but there will be moments of excessive worry about minute things seemingly triggered by nothing. He complains of not being able to breathe at moments and has a "tight feeling" in his chest. After these brief moments, the patient states that feels fatigued. Which of the following should be your lowest priority to order? CBC EKG FBG TSH Complete blood count Dave,a 72 year old patient, receives a score of 22 on a mini mental status exam. What does his score indicate? 51 | P a g e Mild cognitive impairment Which of the following is least likely to be used to treat patients wit depression due to its side effects and high overdose potential? Alprazolam Memantine Citaloprm Phenelzine Phenelzine (Monoamine oxidase) In general, depression must be present for at least how long to qualify for a dysthymia diagnosis? 2 years Short term memory loss is typically an early sign of Alzheimer's disease A patient experiences tremors, impaired swallowing, and drooling, in addition to testing positive for Myerson's sign. Based on the above symptoms and lab results, you conclude that the patient is exhibiting the initial onset of Parkinson's disease. A patient has just been diagnosed with multiple sclerosis. Your colleague is informing her about the different forms of management and mentions beta interferons, immunosuppressive, and plasmapheresis as viable methods. Which is not a method of managing this disorder? Anticoagulants Jessica, a 34-year-old female, presents with episodic, throbbing headaches that last for multiple hours at a time. She tells you that these headaches are usually accompanied by nausea and vomiting. Given the type of headache the patient is most likely experiencing, all of the following visual disturbances are usually zigzag of lights, seeing stars, luminous hallucinations An adult comes to your practice with complaints of occasional weakness, numbness, and double vision but no reported breathing difficulties. You order lab tests and the results indicate lymphocytosis, an elevated protein level in the cerebrospinal fluid, and an elevated igG index level in the cerebrospinal fluid. Which would be most appropriate for managing the patient's most likely condition at this time? Refer the patient to neurology for suspected multiple sclerosis Doug, a 58-year-old male, has recently been diagnosed with cluster headaches. He had previously assumed that he was experiencing migraines and wants to know what the two types of headaches have in common. You tell him that cluster headaches and migraines may share which precipitating factor? Alcohol ingestion 52 | P a g e Sarah, a 26-year-old female, experiences recurring episodes of headaches that are characterized by unilateral and lateralized throbbing. She says that changes in weather and emotional stress tend to increase the incidence. Which of the following diagnostic tests would you least likely order, based on the patient's suspected condition? a. Erythrocyte sedimentation rate b. Computed tomography scan c. Electroencephalography d. Venereal Disease Research Laboratory test Electroencephalography A patient with a seizure disorder as he suddenly stops talking, sits down and stares straight ahead. During the seizure, which lasts approximately two minutes, he makes chewing movements and repeatedly pulls at his shirt. After the seizure, the patient cannot recall the onset of the seizure or describe what happened to him. What type of seizure should the practitioner document? Complex partial On OLD Olympus Towering Tops, A Finn And German Viewed Some Hops CN1: Olfactory: Smell CN 11: Optic: Vision CN III Oculomotor: Most EOMs, opening eyelids, pupillary constriction CN IV Trochlear: Down and inward eye movement CN V Trigeminal: muscle of mastication , sensation of face, cornea, scalp. Think cotton wisp to test corneal reflex CN VI Abducens: lateral eye movement CN VII Facial: Move face, close mouth and eyes, taste ( anterior 2/3), saliva and tear recreation . Test think puff checks CN VIIIAcoustic: hearing and equilibrium CN IX Glossopharyngeal: gag reflex, carotid reflex, swallowing, taste, phonation (one-third) CN X Vagus: Talking, swallowing, carotid reflex CN XI Spinal Accessory: shrug shoulders CN XII Hypoglossal: Move lounge The practitioner knows that which of the following drugs should be increased to help alleviate Parkinson tremors? Benztropine (anticholinergic) You are treating four patients who experience recurring headaches. Which of the following patients is most likely to be experiencing cluster headaches? 55 | P a g e c. Flashing lights d. Impaired swallowing Impaired swallowing The nurse practitioner should tell her that female incidence of myasthenia gravis peaks approximately: In the third decade of age You are treating a patient who is unable to move the right side of her face. As a nurse practitioner, you know that while this condition may resolve without treatment, some patients may benefit from treatment. Which of the following treatment options is least likely to be beneficial a.Prednisone b. Artificial tears c. Pyridostigmine(Used for MG) d. Acyclovir Pyridostigmine(Used for MG) Josephine, age 63, experiences severe pain in her face, claiming that it feels like "an electric shock." In discussing her history, she mentions that her gait has become stiff and unpredictable as of late, and reports experiencing blurred vision. Which of the following is the most likely diagnosis? Trigeminal neuralgia Sandra, a 44-year-old, experiences a unilateral, lateralized throbbing headache with accompanying visual disturbances at least three times a month. She says that these headaches are usually preceded by auras. Of the following medications, which would be the best option to order for prophylactic therapy for the most likely type of headache? Propranolol Your patient starts to have difficulty moving his extremities. He is coughing, wheezing, and having difficulty breathing. You look at his fingers and notice they are starting to turn blue. Which test would you order to distinguish between a myasthenia crisis from a cholinergic crisis? Edrophonium test Which of these patients would be most likely to be affected by a cluster headache? A 55-year-old male You are testing a patient who you suspect has Parkinson's disease. You repeatedly tap the bridge of her nose. Which of the following responses or results would indicate a neurological disorder? Sustained blinking (Myerson's sign) John, age 46, comes to your office with a history of headaches. He states that the headaches last for several hours and feel like a vise is gripping his head, yet he experiences no changes in movement or sensation. Given the most likely type of headache, where would you expect the headaches to be the most intense? 56 | P a g e Neck A patient presents with an inability to move the left side of his face. The nurse practitioner identifies the cause as an inflammatory reaction involving the facial nerve. Which medications would be best suited for treating Bell's palsy Prednisone Susana, age 60, exhibits slow tremors that are exacerbated when she experiences high levels of stress. She also complains of difficulty swallowing and increased muscle rigidity that has developed over the last 5 years. Upon tapping over the bridge of the patient's nose, you note repeated blinking of the eyes, which you recognize as Myerson's sign. which deficiencies does Susana most likely have? Dopamine Sally, a 27-year-old female, reports to your clinic with complaints of recurring headaches.When you ask her to describe the nature of the pain, she says: "I don't know, it just hurts." You decide to ask her about signs and symptoms that accompany the headaches. All of these statements would suggest Sally is experiencing cluster headaches except: a. "It feels like my nostrils just close up. "b. "I keep crying and I don't know why. "c. "My nose just starts running like a waterfall. "d. "I feel like the light starts stabbing at my eyes." "I feel like the light starts stabbing at my eyes. You have just transferred a patient showing classical findings of a transient ischemic attack to emergency care. You know that, in the course of imaging the condition, your patient will likely first undergo a computed tomography scan. Why would this diagnostic be performed? To rule out the possibility of hemorrhage Sonia, a 23-year-old female, has lost the ability to move her eyes downward and inward. Which cranial nerve has been affected? Trochlear CN IV In what area of the brain do childhood tumors predominately occur? Infratentorial region Which cranial nerve controls hearing and equilibrium? CN VIII The practitioner knows that which of the following prophylactic drugs is not used in pediatric migraine management? a. Doxepin b. Bisoprolol c. Ibuprofen d. Amitriptyline 57 | P a g e Doxepin (tricyclic antidepressant) Which of the following characteristics is not typically associated with febrile seizures? a. Fever with rectal temperature of 102 °F or above b. Generally manifests in early infancy by 3 months of age c. There is usually a family history of febrile seizures d. There is typically a loss of consciousness with this type of seizure Generally manifests in early infancy by 3 months of age. Febrile seizures do not generally manifest before 6 months of age Tension and psychogenic headaches are commonly associated with what mechanism of headache pain? Muscular contraction Jamie, a 4-year-old male, presents to the clinic in a sleepy, afebrile state. His mother mentions that, after his afternoon nap, he started screaming in their brightly-lit den and began vomiting. When she tried to talk to him, he spoke nonsense. When you talk to Jamie, he rouses, exhibits normal speech, and says he feels fine. What is the most likely source of Jamie's symptoms? Confusional migraine A patient has been referred to your practice by an optometrist, who found Lisch nodules in the patient's irises. Which finding would best help to confirm the suspected diagnosis of neurofibromatosis type I ? Axillary freckling Which of the following best defines a simple partial seizure? Minor motor symptoms with no loss of consciousness Which of these signs would indicate meningitis in a patient? Kernig's sign Pain upon extension of the leg when the hip is flexed 90° In a patient with tic disorders, all of the following presentations would typically be recognized as a simple tic except: 60 | P a g e to meet an approaching deadline. Based on these findings, which of the following would you most likely suspect as a cause of the patients's most likely condition? Anxiety A male patient presents with a noticeable yellow tint to his skin. In the workup, which of the following lab values would most likely be of priority to be assessed? Total bilirubin You are prescribing medication for Helicobacter pylori eradication to a 44 year old female with NSAID- induced peptic ulcer disease. Considering the fact that the patient must still remain on a chronic, daily NSAID regimen, which of the following drugs should you strongly consider including as part of the patient's prophylactic therapy? Misoprostol You are ordering tests to confirm a diagnosis of bowel obstruction for a 36 year old male who presents with complaints of committing, inability to pass stool, bloating, and severe abdominal pain around his "belly button." Which of the following indications would provide the earliest suggestion that the patient's condition may be due to colon cancer? Positive guaiac test Which of the following serology markers indicates a previous exposure to hepatitis A? IgG You are prescribing a patient an initial trial of H2 receptor blockers for management of peptic ulcer disease. When should these be orders? At bedtime You are consulting Amber, a 41 year old female who has gastroesophageal reflux disease. In discussing her diet, you should advise her to avoid consumption of all off the following food products except: a. Caffeine b. Spices c. Peanut butter d. Alcohol Peanut butter You are evaluating a 45 year old male who presented to your clinic with a low-grade fever, itching, and pain in the right upper area of his abdomen. He mentions during the evaluation that he has noticed his stool is "really pale, like kind of grey". Based o the patient's signs and symptoms, which of the following lab results is least likely to help in confirming a diagnosis? Elevated white blood cell count Which agent is used specifically for the treatment of peptic ulcer disease and may also use as an alternative drug for H2 receptor blockers to manage gastroesophagal reflux disease? 61 | P a g e Omeprazole You notice on your patient's charts that he is dealing with a recurrent case of ulcerative colitis. Given your knowledge of ulcerative colitis, which of the following symptoms would you most expect your patient to report? Bloody diarrhea As a nurse practitioner, you know that a diagnosis of diverticulitis usually involves a physical exam, imaging studies, and which of the following lab findings? Elevated WBC count, elevated ESR You are performing a health exam on James, a 54 year old patient with a long history of alcohol addiction. You notice that his abdomen is swollen and his skin is a slight shade of yellow. When you ask him about his diet, he tells you that he does not normally have an appetite because he usually feels nauseous and his stomach "has been bothering [him] lately". Suspecting hepatitis resulting from his history of alcohol abuse, you know that all of the following options would be usually used in the patient's treatment plan except: a. Mesalamine b. Vitamin K c. Oxazepam d. Folic Acid Mesalamine You are discussing treatment options for a patient with severe case irritable bowel syndrome. What three medications would you most strongly consider to help manage this condition? Anticholinergic agents, antidiarrheal agents, and antidepressant agents Which of the following patients has the lowest risk of developing colon cancer? a. A female whose aunt and grandmother both developed ovarian cancer b. A hispanic male with a history of peptic ulcer disease c. A female with a high-fat diet that includes red meat and refined carbohydrates d. A male with a history of recurrent inflammatory bowel disease A hispanic male with a history of peptic ulcer disease A 56 year old patient presents with complaints of nausea, vomiting, and watery diarrhea, all of which have lasted for 4 days. Upon examination, you note hyperactive bowel sounds, abdominal distension, and tachycardia. Based on these findings, which of the following lab tests would best help to diagnose the patient's condition? Stool test Which of the following indications would the nurse practitioner most likely expect to see during a physical exam that would suggest a diagnosis of cholecystitis? Pain upon inspiration while fingers are placed under the right rib cage 62 | P a g e Which test is used to differentiate prior exposure to the hepatitis C virus from a current infection? Polymerase chain reaction Which of these statements would most likely suggest that your female patient was predisposed to the hepatitis A virus? "I recently returned from a trip to Haiti doing hurricane relief work. Had some good oysters over there" Owen, a 45 year old male, presents to the clinic complaining of watery diarrhea, stomach cramps, and vomiting. He states that he has been under tremendous pressure at work and is so stressed out that the often feels nauseous. Based on these findings for the most likely diagnosis, which of the following indications would you least expect to find during a physical examination? Left lower quadrant pain A 52 year old male presents with complaints of heartburn and a persistent "stomahc ache" that has increased in intensity over the past 2 weeks. He points to his upper abdomen to indicate the location of the pain, and he says that he usually "feels better" after he eats. These findings most strongly suggest: Duodenal ulcer Casey, a 28 year old male, is admitted to the emergency room after complaining of a sharp pain in his right side and of a fever. He says that he noticed he woke up feeling nauseous the day priod, and that the pain slowly started around his "belly button" and shifted to his right side slowly throughout the day. What complication is Casey most at risk of developing if the patient's condition were to be left untreated any longer? Gangrene Which of these statements is false in regard to ulcerative colitis and Crohn's disease? a. Ulcerative colitis and Crohn's disease may occur at any age in both males and females b. Crohn's disease may affect the mouth c. Ulcerative colitis can occur in any part of the gastrointestinal tract. d. Ulcerative colitis and Crohn's disease both have unknown etiologies Ulcerative colitis can occur in any part of the gastrointestinal tract. False it is typically limited to the large colon Which of the following indications of bowel obstruction is properly paired with the source of obstruction? Minimal abdominal distention; proximal obstruction What 2 gastrointestinal disorders are more common in women that in men? Diverticulitis and irritable bowel syndrome H. pylori triple therapy Clarithromycin + Amoxicillin + PPI (CAP) 65 | P a g e Fitz-Hugh-Curtis syndrome (PID) A patient presents with a new onset of jaundiced sclera, right upper quadrant tenderness, and anorexia. The nurse practitioner draws labs with the following results: AST = 24 mg/dL; ALT = 13 mg/dL; HbsAg = negative; anti-HBc = negative; anti-HBs = positive. How will the nurse practitioner interpret these data? The patient has immunity from hepatitis B vaccinations A patient diagnosed with irritable bowel syndrome (IBS) tells the nurse practitioner that over the past few months, she has been experiencing frequent bouts of constipation. Which prescription will the nurse practitioner add to the treatment plan? Lubiprostone (Amitiza) 8 mcg PO BID A patient recently returned from a trip to Africa and is experiencing 10 to 12 loose stools/day. The patient takes metformin every morning and otherwise is in good health. Which medication will the nurse practitioner prescribe to treat the diarrhea? Ciprofloxacin (Cipro) in a single dose A mother brings her 1-month-old infant to the clinic for evaluation, stating the infant has poor weight gain despite constant hunger and projectile vomits after eating. The nurse practitioner notes that the infant has a distended abdomen. Upon palpation, the nurse practitioner detects an olive-shaped mass in the epigastrium. The nurse practitioner will: Refer the child for ultrasound testing because the infant presents with signs and symptoms of pyloric stenosis: abdominal distention, dehydration, projectile vomiting, and failure to thrive. The span of the normal adult liver is: 6 to 15 cm in the midclavicular line The most important clue for ulcerative colitis is bloody stools that are covered with mucus and pus along with the systemic symptoms fatigue, low-grade fever. A mother brings her 2-year-old to the clinic with nausea and vomiting, fever, and abdominal pain. The mother noticed blood in the child's diaper this morning. Upon physical examination, the nurse practitioner palpates a nontender abdominal mass extending from the flank toward the midline. Swelling is noted on the side of the mass, and the child has an elevated blood pressure. Which diagnosis is most likely? Wilm's tumor (Does not cross the midline) 66 | P a g e All of the following deficiencies are common in pediatric patients with celiac disease, except: Folate Ferritin Vitamin D Vitamin B12 Vitamin D Periumbilical to right lower quadrant pain occurs more frequently in CD, and left lower quadrant pain is more classically associated with ulcerative colitis (UC). All of the following assessment findings are associated with Crohn's disease, except: Mass palpated in abdomen Weight loss and dehydration Fistula formation Inflammation limited to mucosal layer of colon Inflammation limited to mucosal layer of colon Which assessment finding is expected in a 3-year-old male with penopubic epispadias? Urine leakage with stress Lauren, a 29 year old female, presents to the clinic with a sudden onset of productive cough that is accompanied by wheezing and a high-grade fever. During the physical exam, you find no evidence of lung consolidation, as the lungs are clear to auscultation and resonant to percussion. Given the most probable condition, which of the following pathogens would be the least likely cause? a. Streptococcus pneumoniae b. Neisseria gonorrhoeae c. Mycoplasma pneumoniae d. Haemophilus influenzae Neisseria gonorrhoeae A left-hand-dominant high-school tennis player presents with a concern of left elbow pain. Which signs and symptoms suggest that the patient is suffering from tennis elbow? (Select all that apply.) Lateral elbow pain with tenderness at the lateral epicondyle Elbow pain with grasping movement Which treatment is recommended for an infant with noncommunicating hydrocele? 67 | P a g e Reassess in 1 year Patients with medial epicondylitis (golfer's elbow) typically present with: Elbow pain that worsens with pulling activity Medial elbow pain with tenderness at the medial epicondyle Decreased grip strength. A patient presents with epigastric pain and intermittent diarrhea that worsens with food. She has a history of multiple ulcers. The nurse practitioner reviews the results of a serum fasting gastrin level and finds there is no inhibition of gastrin levels. Which diagnosis is most likely? Zollinger-Ellison syndrome A 70-year-old man with open-angle glaucoma is prescribed timolol (Betimol) ophthalmic drops. All of the following are contraindications to Betimol ophthalmic drops, except: Overt heart failure or sinus bradycardia History of asthma Second- or third-degree atrioventricular (AV) block Migraine headaches Migraine headaches A patient presents for follow-up with the nurse practitioner after receiving a diagnosis of acute diverticulitis. The nurse practitioner is educating the patient about how to avoid complications. What instructions should be included? (Select all that apply.) Maintain a high-fiber diet. Take amoxicillin-clavulanate 875/125 mg BID as ordered Take a laxative every other day Use psyllium fiber supplementation daily Report fever to healthcare provider immediately Take amoxicillin-clavulanate 875/125 mg BID as ordered Report fever to healthcare provider immediately A 55-year-old male patient with a body mass index (BMI) of 30 has a history of angina and type 2 diabetes. His lipid profile results are total cholesterol of 280 mg/dL, low-density lipoprotein (LDL) of 195 mg/dL, and high-density lipoprotein (HDL) of 25 mg/dL. The nurse practitioner diagnoses him with hyperlipidemia and wants to start him on statin therapy. What intensity of treatment is recommended for this patient? High-intensity statin Which condition will the nurse practitioner monitor for in a geriatric patient taking hydrochlorothiazide and nifedipine (Procardia XL)? Acquired neutropenia (Hydrochlorothiazide can cause bone marrow suppression) A 73-year-old male patient presents with bilateral edema in the legs and feet, weight gain of 15 lb, and decreased urine output. The nurse practitioner will order: 70 | P a g e Ethambutol may be dropped Victor, a stocky 40 year old male, with complaints of difficulty breathing and "endless amounts of gunk whenever [he] cough[s]." During the visit, he coughs up a substantial amount of yellow phlegm. A blood test reveals an increased hematocrit level, and a physical examination detects lungs that are normal upon percussion. Given the most likely condition of Chronic bronchitis, which of the following findings would you expect? decreased forced expiratory volume in 1 second increased total lung capacity Increased functional residual capacity Increased residual volume An HIV positive patient develops a low grade fever. During his visit, he complains of fatigue, a reduced desire to eat, and a dry cough. He coughs in front of you, producing sputum that is tinged red. You order a chest c-ray, which reveals a small homogeneous infiltrate in the upper lobes. Given the most likely condition, what combination of drugs would be most effective for treatment? isoniazid, rifampin, ethambutol, pyrazinamide (9months) Which of the following findings would be indicative of chronic obstructive pulmonary disease? a. Hyperinflated lungs b. flattened diaphragm c. Reduced FEV1 d. Increased residual volume We have an expert-written solution to this problem! What is first-line treatment for asthma inhaled corticosteroids What is the preferred reliever medication for asthma according to the Global initiative for Asthma (Gina, 2020) treatment guidelines? low-dose ICS with formoterol (ICS-LABA) Which of these characteristics would lead you to believe she has emphysema and not chronic bronchitis? Thin and wasted habitus In cases of asthma, the trachea and bronchi typically become more: Responsive 71 | P a g e Winston, a 42 year old male, is an HIV-positive patient whose tuberculosis skin test returns with an elevation of 5 mm. After confirming a diagnosis of TB, you prescribe a traditional drug regimen. For what minimum period of time is Winston expected to continue his regiment? 9 months A patient presents to you with a productive cough and headache. The patient displays no other signs or symptoms and is afebrile. Upon physical examination, you note that his lungs are clear upon auscultation and resonant upon percussion. You suspect acute bronchitis; as the indications are far from definitive, however, you wish to rule out other conditions. Which diagnostics would be least helpful in confirming or excluding probable conditions at this time? a. Sputum culture b. Chest x-ray c. Sputum sensitivity d. Pulmonary function test Pulmonary function test Jordan, a 45 year old male, comes to the clinic with a dry cough, weight loss, night sweats, and fatigue. A chest x-ray reveals small, homogeneous infiltrates in the upper love of the right lung. Give the most likely condition, which of the following tests are you most interested in ordering to confirm the most likely diagnosis? a. Complete blood count b. Acid-fast bacilli smear c. Liver function d. Serum creatinine Acid-fast bacilli smear A 29 year old dental assistant returns to your practice to have her annual purified protein derivative test read. The nurse in your office documents an 8 mm induation that is reddened. Which of the following is your best plan of action? a. Order an acid-fast bacilli test b. Send the patient for a chest x-ray c. Order a complete blood count d. Have her return for testing in 1 year Have her return for testing in 1 year Samuel, a 45 year old male, presents to the clinic with a cough, headache, excessive sweating, fever, sore throat, and soreness in the chest. A chest x-ray reveals the presence of infiltrates. Given the most likely diagnosis, which atypical pathogen that may have caused his condition? Chlamydophila Mycoplasma Legionella Iris, a 32 year old patient, presents with fever, shaking chills, and malaise. A physical examination reveals lung consolidation, purulent sputum, and an increased fremitus. Further diagnostic evaluation shows a low white blood cell count and infiltrates on a chest x-ray. Give the most likely diagnosis, which medication would be best suited for treating Iris's condition? Amoxicillin or Doxycycline or azithromycin All of the following would be consistent with a typical manifestation of severe asthma except: a. Respiratory rate of 35 bpm b. Pulse of 125 bpm c. Pulsus paradoxus of 15 mmHg d. White blood cell count of 1800 eosinophils/mcl Pulsus paradoxus of 15 mmHg (moderate is 10-20) 72 | P a g e Ethel, a 90 year old female, presents to the clinic with fever, headache, excessive sweating, and soreness in the throat and chest. Results from a complete blood count reveal a low white blood cell count an a chest x-ray shows infiltrates. Based on the most likely diagnosis, which antibiotic should Ethel be treated with? Levofloxacin You order a regimen of antibiotics to treat tuberculosis, which includes 15 mg/kg of ethambutol. With this medication as part of the regimen, which of the following conditions should the patient be tested for ? visual acuity and red-green color perception An otherwise healthy 45 year old patient with atypical pneymonia would best be treated with which medication? amoxicillin, doxy, or azithromycin which of these conditions are restrictive disorders? Pulmonary fibrosis Tuberculosis Heart failure Kyphoscoliosis obstructive lung disease example Bronchiectasis, COPD, asthma, cystic fibrosis Julia, a 19 year old female, comes to the clinic with a cough, headache, sore throat, and excessive sweating. After ordering some diagnostic measures, you find an elevated white blood cell count and infiltrates in the lungs. Which condition is Julia most likely experiencing? Atypical pneumonia Percy, a 38 year old male, comes to your practice with fever, cough, headache, sore throat, and excessive sweating. After ordering a series of labs and diagnostics, you find that he has an elevated white blood cell count and chest infiltrates. Given the many ways the microorganism responsible for the most likely condition might gain access to the host, which route for infection is least likely? a. Aspiration b. Inhalation c. Exogenous penetration d. Hematogenous dissemination Exogenous penetration The practitioner recognizes that all of these are pulmonary findings in the geriatric patient: Total lung capacity unchanged Residual volume increases Vital capacity decreases The number of mucus-producing cells increases 75 | P a g e Intermittent Step 1 A patient presents to the office with complaints of asthma symptoms daily with occurrences of nighttime asthma symptoms twice weekly. Forced expired volume in one second (FEV1) is >79%, and respirations are 20 breaths/minute. What asthma classification protocol should the nurse practitioner follow? Step 3 moderate persistent An infant who does not have a history of reactive airway disease and allergy has both inspiratory and expiratory wheezing accompanied by fever and profuse, clear nasal discharge. Which of the following is most likely? Bronchiolitis (RSV) Which of the following antihypertensive medications should the nurse practitioner avoid when treating patients with emphysema? Beta-blockers Which of the folding regimens would best constitute step 3 moderate persistent in the stepwise management of a patient with asthma ICS-LABA such as budesonide-formoterol How frequently would a typical pediatric patient with mild persistent asthma use a rescue inhaler more than 2 days a week Which of the following antibiotic regimens is recommended as first-line treatment for community- acquired pneumonia in patients with no comorbidity? Doxycycline 100 mg PO BID Peak expiratory flow is calculated using which factors? (Select all that apply.) Height, Age, Gender Think HAG A adult student is seen in the school health clinic with complaints of a hacking nonproductive cough, rhinorrhea, pharyngitis, and malaise for the past 2 weeks. He does not take any medications, denies any allergies, and has no significant medical history. Physical examination reveals a low-grade temperature of 99.9°F, respirations of 16 breaths/min, pulse of 90 beats/min, and scattered rales and wheezing of the lungs. The patient does not appear toxic. The total white blood cell count is 10,500/µL. Which diagnosis is most likely? Atypical pneumonia. The most common organism causing community-acquired atypical pneumonia is Mycoplasma pneumoniae, A chest radiograph shows an area of consolidation on the lower lobe of the lung. Which of the following conditions is most likely? 76 | P a g e Bacterial pneumonia The primary function of all of the following pharmacologic agents is to treat inflammation in the lungs, except: Nedocromil sodium inhaler (Tilade) Cromolyn sodium inhaler (Intal) Theophylline oral (Theo-24) Fluticasone inhaler (Flovent) Theophylline oral (Theo-24 bronchodilator) A premature infant should typically be given palivizumab for the prevention of RSV bronchitis if gestation is less than 29 weeks Work up for cystic fibrosis includes pilocarpine iontophoresis sweat test pulmonary function test Sputum culture What is the most common etiology of pneumonia in children 1-3 streptococcus pneumonia You are assessing for egophony in a patient with suspected pneumonia. You ask the patient to produce a lone E sound. What should should you anticipate to osculate when the patient has lung consolidation? A long A or ay sound What pathogens responsible for pneumonia typically demonstrate lobar consolidation as a radiographic characteristic STREPTOCCOUS PNEUMONIAE Hamemophilus influezae Klebsiella pneumonia What two laboratory finding best characterizes the findings of restrict disease in pulmonary function test primary reduced volumes and reduced expiratory flow rates A 7 year old with asthma symptoms once per day with 2-3 nights per week awakening. How would you classify asthma? Moderate persistent Your colleague's patient experiences a sudden onset of pain in her lower leg. The area where the patient identifies the pain is erythematous and warm to the touch but not swollen. The patient also has a consistent temperature. Your colleague suggests elevating the leg and applying warm compresses at night. He also prescribes a nonsteroidal anti-inflammatory drug and suggests bed rest for at least a week 77 | P a g e to help the pain subside. Based on these findings, which of these measures is not a correct form of treatment for the patient's condition? a. Elevating the leg b. Applying warm compresses c. Prescribing non-steroidal anti-inflammatory drugs d. Ordering bed rest Ordering bed rest Which of the following is not a common finding of acute left-sided heart failure? a. Coarse rales in all lung fields b. Hepatomegaly c. Wheezing, frothy cough d. An S3 gallop sound Hepatomegaly With regard to S3 and S4 heart sounds, which statement is true? An S3 sounds like "Kentucky" and is expected during pregnancy but otherwise, it is not a normal finding in the adult. Which of the following statements is typical of the heart murmur with which it is associated? V/VI, Very loud murmur can be heard off the chest wall with one corner of the stethoscope. You hear a low-pitched diastolic rumble at the fifth intercostal space with the patient in the left lateral position. The murmur does not radiate. You refer the patient to cardiology to "rule out" Mitral stenosis Which type of heart sound is caused by aortic and pulmonic valve closures? S2 Frank, age 56, has just been diagnosed with hypertension. After looking at his medical history, you decide that propranolol would be the most effective form of treatment, as it would treat his hypertension while also managing other conditions. Given the decided treatment, Frank most likely has which of the following co-occurring conditions? Migraines According to the Eighth Joint National Committee hypertension guidelines, which of the following is used to estimate 10-year and lifetime atherosclerotic cardiovascular disease risks? High-density lipid cholesterol, Race, Diabetes status, systolic BP, race, total cholesterol, and history of smoking. Dependent rubor is a physical finding associated with which of these diseases or conditions? Peripheral vascular disease 80 | P a g e ST-segment depression in leads I and aVL indicates which type of infarction? inferior wall An anteroseptal wall infarction would present with Q waves in leads V1-V4. Richard, age 55, presents with pain and tenderness in his left ankle while walking, as well as a low-grade temperature. A physical examination finds edema around Richard's ankle. Which of the following conditions should you most suspect? Deep vein thrombosis A patient has a negative D-dimer test. Which of the following conditions would this lab result most strongly rule out? Deep vein thrombosis During a routine physical for Leonard, age 56, you note a diastolic murmur. The murmur presents with a blowing at the second left intercostal space. He most likely has which valvular disease? Aortic regurgitation Which of the following patient findings constitute the strongest indication for pharmacologic revascularization? Chest pain that has lasted for 3 hours, an ST segment elevation of 0.3 mV in the precordial lead, followed by an ST segment elevation of 0.3 mV in lead I You are examining Victor, age 67, and identify a heart murmur. This murmur is fairly loud and can be heard with one corner of the stethoscope off the chest wall. You are most likely listening to which of the following murmurs? Grade IV/VI Which statement is true statement regarding hypertension? A classic presentation of hypertension includes a suboccipital, "pulsating" headache, usually in the morning, which resolves throughout the day. Benjamin, age 62, presents with wheezing, a frothy cough, and shortness of breath at rest. The cardiologist's report reveals that his condition derives from the inability of the heart to contract, which results in decreased cardiac output. Which of these is Benjamin most likely experiencing? Systolic acute Diastolic heart failure also results in decreased cardiac output; however, this is a result of the heart's inability to relax and fill 81 | P a g e You are examining a chest x-ray of a patient with chronic right-sided heart failure. Which of the following would you expect to find? Kerley B lines, Pulmonary edema, Pleural effusions You are performing a fundoscopic exam on George, age 55, who presents with changes that include flame-shaped retinal hemorrhages, superficial white areas with feathered edges, and a swollen optic disc. Which of the following conditions is he most likely experiencing? Malignant hypertension As a nurse practitioner, you know that which two cardiac enzymes are entirely cardio selective and only elevate when myocardial damage is present? CK-MB and troponin I The practitioner is considering antihypertensive therapy for a patient. Which statement is most accurate? Angiotensin-converting enzyme(ACE) inhibitors may cause vasodilation, rash, taste disturbances, hyperkalemia, and renal impairment. You hear a loud pansystolic murmur that is "blowing" at the apex of the heart. You refer the patient to a cardiology service to rule out which of these? Mitral regurgitation Your patient asks, "I really thought I was having a heart attack. How can you tell the difference between angina and having a myocardial infarction?" Which of the following responses would provide the patient the most accurate information? "Pain of angina can usually be relieved by resting or lying down." Sandra, age 65, presents to the clinic with chest discomfort. She describes her pain by clenching a fist to her chest. Upon examination you detect transient S4 sound. Which of the following would be in the treatment plan for Sandra? Nitroglycerin, Atenolol, Diltiazem Peter, a 55-year-old Caucasian man, has a consistent blood pressure reading of 145/90 mmHg. His medical history reveals he was diagnosed with diabetes a few years prior. Which of the following would most likely be the drug class of choice for Peter at this time? Angiotensin-converting enzyme inhibitors Your patient comes to the clinic with a blood pressure reading of 184/112 mmHg, but is otherwise asymptomatic. Your clinic does not have the appropriate medication to treat his condition, so you refer him to the local ER. Which medication would you most expect your patient to receive at the ER? Clonidine 82 | P a g e Which of the following supplements is used to potentially slow cognitive decline in AD? A. vitamin B12 B. vitamin E C. ginkgo biloba D. St. John's wort vitamin E To rule out Kawasaki disease, which test would best help to isolate the child's most likely condition? Erythrocyte sedimentation rate Which of the following types of congenital heart defects is most likely to present with right to-left shunting? Tetralogy of Fallot Which of the following diagnostics is least likely to indicate a diagnosis of rheumatic fever? a. Electrocardiogram b. Aldosterone level c. Echocardiogram d. Throat culture Aldosterone level How would you best auscultate to confirm a venous hum? Have the patient sit up; auscultate the right upper sternal border. What feature best distinguishes the characteristic murmur of patent ductus arteriosus? A "machinery" noise at the left upper sternal border Which of these heart sounds would most strongly indicate a cardiac disorder in a pediatric patient? S4 Which of the following types of heart murmurs is not typically considered an innocent heart murmur? Radiating murmur During auscultation of a newborn, a grade IV murmur is heard with a holosystolic thrill at the left lower sternal border. You suspect a congenital heart defect and order an x-ray. What findings would be most consistent with a suspected diagnosis of ventricular septal defect? Cardiomegaly with increased pulmonary vascular markings Rib notching due to collateral circulation which also commonly presents with cardiomegaly is a distinguishing feature of coarctation of the aorta A boot-shaped heart is a characteristic x-ray finding of 85 | P a g e Which of these scenarios best describes the primary component of malpractice? A nurse practitioner acts hastily in treating an aneurysm, resulting in otherwise avoidable partial paralysis Which of these is covered under the auspices of the Danforth Amendment? A patient's right to refuse care when admitted to a federally- funded institution You are writing a manuscript about an unsolved medical case in which the patient was being treated for heartburn and unexpected died. Which of the following would best describe this type of research? Qualitative The privacy Rule of the HIPAA establishes that a patient whose rights are being denied or whose health information is not being protected can file complaints with certain entities. Which of the following is not one of these entities? a. Health insurers b. Healthcare providers c. U.S. government d. State agencies d. State agencies You are performing a research study to determine the effects of smoking. You collect data from two sample groups. The first group consists of smokers between the ages of 21 and 45, and the second group consists of smokers between the ages of 65 and 80.which type of research does this study best exemplify? cross-sectional What purpose does Cronbach's alpha typically serve in a statistical research analysis? Determines correlation of values in a survey instrument to assess its reliability a male patient who was admitted to the hospital is diagnosed with a highly contagious, viral illness that could be fatal and potentially cause an epidemic. The decision to keep him in isolation and limit the number of health care providers who come into contact with him would most closely demonstrate which ethical principle? utilitarianism Which of the following best delineates the degree to which a nurse practitioner may prescribe medication to patients? Nurse practice acts Which entity is required to comply with HIPAA guidelines? value-added networks and other health care clearinghouses 86 | P a g e James, a 52 year old male, has just fallen into a diabetic coma. Robert, his domestic partner arrives at the hospital with a living will from James regarding theses particular circumstances. Which component of this will would grant it the strongest standing? The will names Robert as proxy and grants him authority to do as he sees fit in maintaining James's care. Power of attorney must usually meet a criteria before it will be honored by most institutions? The document must form a part of the patient's health care directive Which diabetic med should be avoided in patient with gastroparesis liragkutide (victoza) What is the standard purpose of a t-test? to evaluate the differences between two groups A nurse learns that a female patient's injuries stem from a case of domestic violence. What legal steps should the NP take in this case? There is no legal obligation to do anything What is an example of battery angrily striking a patient on the shoulder when he or she will not stand still What law sets national standards for the security of electronic protected health information? Health insurance portability and accountability Respect and spiritual needs are considered components of which principle that the NP must often consider in therapeutic communication with a patient? cultural competency What statement is true regarding licensure it is acquired through rules and regulations established by governmental body In the course of your research study, you determine the measure of the interdependence of two random variables to be -1. What research term best describes the value? negative correlation What percentage of the patient's bill for physician services is typically covered by medicare 80% The patient safety work product often contains specific findings, procedures, and management methods recorded during a safety event nurse case manager utilizes a comprehensive systematic approach to provide quality care by 87 | P a g e mobilizing, monitoring, and controlling patient resources during illness while balancing the quality and cost of resources. For home NP visits billable for Medicare B services, an NP usually does not need a physician's order to bill under the NPs own provider number unless The NP is proving nursing services exclusively What agency enforces the protections safeguarded by the health insurance portability and accountability act The office for civil rights a care map identifes all of the following points relevant to the management of a condition Guidelines for planning and managing care delivered by all disciplines, identifies common patient- related problems of a specific case type, and sets out day to day goals that the patient must achieve as well as the final desired clinical outcome Describe a cohort study Compares a particular outcome in groups of individuals who are alike in many ways but differ by certain characteristics. In states that recognize living wills, what condition must be met so that the will is legally revoginzed the will must be specific enough and address the problem at hand Which plan of medicare not only offers coverage for prescriptions but also requires a copay for each as well as a monthly premium medicare D What scenario most effectively demonstrates a care of negligence? An NP waits to administer anticonvulsants to a patient experience multiple prolonged unprovoked seizures, in order to see if they stop on their own. All of the following are typical physiological changes in the central nervous system of the elderly population except a. Decreased beta-adrenergic responsiveness (peripheral nervous system) b. Increased alpha responses c. Decreased dopamine receptors d. Increased muscarinic parasympathetic Decreased beta-adrenergic responsiveness (peripheral nervous system) which of the following is attributed to decreased renal function in the elderly with regard to prescription medication Impaired drug elimination 90 | P a g e During an annual physical exam, you notice that 12-year-old Lisa has breast enlargement without seperate nipple contour. You tell her she can expect which of the following in the next stage of her development: Onset of menses A 21-year-old male comes to you complaining of a painful bump with a small, red-looking halo on his penis. The affected area is about 5 mm in diameter, soft, and tender to the touch. He states that the halo has been there for about a week and is painful. chancroid Of the following, which is a necessary guideline for interviewing adolescents in a clinical setting? Excluding the parents from the interview with the adolescent There is concern at your clinic that some children with developmental delays are not being identified soon enough. You are examining a 6-year-old male. Of the following, which is the best tool to identify risk for developmental delay in this child? Denver II A 65-year-old noticed persistent, sharp, and painful spasms in her left cheek. She describes the pain as feeling like "an electric shock." Which would be the least appropriate treatment for this condition? Tricyclic antidepressants Calcium channel blockers Muscle relaxants Anti-seizure drugs Calcium channel blockers Your 57-year-old male patient has awakened nightly for the last three weeks to severe throbbing pain around his left eye, particularly after drinking alcohol. The pain usually subsides within an hour. A physical exam reveals nothing too unusual, other than eye redness and rhinorrhea. Of the following, which would be the best way to manage the patient's condition? Clopidogrel Sumatriptan Over-the-counter analgesics Ticlopidine Sumatriptan After reviewing Betsy's electrocardiography and echocardiogram results, it is clear that her heart is unable to contract sufficiently, resulting in decreased cardiovascular output. Which heart failure does this best describe? Systolic A 27-year-old female comes to your office and explains that she has recently felt an itching and inflammation in her vagina, despite being sexually inactive for several months. Upon physical exam, you 91 | P a g e note vulvovaginal erythema. In addition, you discover a thick, white, curd-like discharge. Which of the following is she most likely experiencing? Candidiasis An elderly man presents with swollen lymph nodes and a painless lump in his neck. He also reports that he recently has been feeling generally weak and sometimes has a fever. During your exam, you note tiny, red spots on the skin of his hands. Which of the following is required to confirm the diagnosis of the most likely cause of his presentation? Bone marrow aspiration A 24-year-old patient comes to your office with a sore throat, nasal congestion, and a cough. The patient repeatedly clears her watery congestion into a tissue. The patient does not have a fever, and a physical exam is unremarkable. Given only this information, what is the most likely cause of her condition? common cold Rosario just gave birth to her first child born in the United States. She and her family, which includes two other children, recently immigrated from Mexico. You compliment her baby for doing well and being adorable. The next day, your supervisor gets an angry call from Rosario's husband regarding your behavior. Which of the following is the most likely complaint? In some Mexican cultures, you must always touch the baby when complimenting him or her. Steven, a 48-year-old father and husband, has been brought to you by his family for alcoholism treatment. Steven, however, denies being an alcoholic. At this point, which of the following questions is the most effective one to ask this patient? Have you ever felt the need to cut down on your drinking? A 35-year-old male presents with complaints of a "strange swelling" in his throat. He also mentions that he has lost about 12 pounds in the past several months. A physical examination reveals that he has swollen lymph nodes, and there is involvement in the right axilla and right side of the neck. Based on the suspicion of lymphoma, which stage of disease does this presentation most likely represent? Stage II A 30-year-old female presents with a cough and headache. She reports that she also has been wheezing for nearly a full week. Upon examination, you note a low grade fever and bilateral rhonchi; however, after coughing, her lungs are relatively clear to auscultation. Of the following, which is the most likely diagnosis? Acute bronchitis For patients with inadequate iron intake, most of what composes a patient's blood will be lower than normal in laboratory findings. What aspect of blood, however, may actually be found to increase? Red cell distribution width 92 | P a g e A child's serology results come back showing Anti-HCV and HCV RNA. Which of the following may be included in his treatment plan? Interferon and ribavirin A 31-year-old patient comes to your office complaining of hearing loss but only in the right ear. When examining her, you notice some erosion on her eardrum in the middle ear. Which of the following is the most likely diagnosis and treatment? Cholesteatoma; surgery Peter comes to you after having stood for four straights days working his company's booth at a convention. He complains of intermittent numbness and tingling on his right foot up into his calf, as well as pain in the calf. During examination, you have Peter walk on the heels of his feet and you note pain and weakness of the dorsiflexion mechanism of the great toe and foot. Which of the following is your most likely suspicion? L4-L5 disk pathology The term "pervasive development disorders" refers to a group of conditions that involve delays in the development of many basic skills. Children with these conditions often are confused in their thinking and generally have problems understanding the world around them. Which of the following is a factor leading to this condition? No known factor A man who recently attempted suicide is brought to your office by his wife. Having conducted a significant amount of research on her own about suicide prevention, she has many questions. Which of following choices is the least effective treatment for her husband? Antidepressants if the situation is chronic Intervention if the patient's risk is escalating Antipsychotics if the patient is psychotic Hospitalization if the situation is acute Intervention if the patient's risk is escalating Burtonian lines can best be described as which of the following? Thin blue-black line on the margin of the gums Autism in children is generally identified by 3 years of age. Alice is 30 months of age, and her mother mentions that she is not playing with others at preschool, is unable to pretend play, and has a very limited vocabulary. Her only interest is stacking blocks, at which she is very good. When considering the disorders in the spectrum of pervasive development disorders, which of the following should be your initial consideration in your differential diagnosis? Autism 95 | P a g e information that pertains to Brett's experience with the healthcare system. What is not true about HIPAA? All sharing of a patient's information requires written authorization. The measles, mumps, and rubella (MMR) vaccine must be administered in two doses. When should the MMR vaccines typically be given? First dose: 12 to 15 months old; second dose: 6 years old A 25-year-old male presents with a bubo in the right groin, as well as stiffness and aching near his genitals. You note that there is swelling in his right inguinal area. You aspirate the bubo and prescribe doxycycline. For what condition would this be the proper treatment? Herpes simplex virus type 2 Genital warts Molluscum contagiosum Lymphogranuloma venereum Lymphogranuloma venereum A 56-year-old marketing executive has various past blood pressure (BP) readings of between 140 to 150 mmHg (systolic) and 80 to 100 mmHg (diastolic) over the last few months. During an office visit, you find his current BP is 142/88 mmHg. This collective data confirms a diagnosis of hypertension as per the ACC/AHA guidelines. Based on these guidelines and the patient's current blood pressure, how would you classify his hypertension today? Stage II Melanie, an 8-month-old baby, comes to the clinic with bluish skin and a cleft palate. The pediatric nurse practitioner (NP) notes that the baby has twitching around her mouth with spasms on her hands and arms. Melanie's parents also report seizures. Upon further observation and interview, the NP learns that the baby is also developmentally delayed in many areas. Which of the following should be a part of your differential diagnosis? DiGeorge syndrome A mother brings her 5-month-old son for a checkup. She is concerned that her son's right leg appears shorter than his left. Furthermore, she states that his right leg looks as if it were turned outward. She adds that it does not seem as if her child is in any pain though. Suspicious, the nurse examined the child by adducting his right hip while applying pressure on the knee. For which of the following conditions is the nurse practitioner examining? Hip dysplasia A 30-year-old female presents to your office with frequent and debilitating headaches that are particularly intense about her neck. She says that her neck and shoulders feel extremely tight and tense during these headaches. However, she denies any neurologic deficits. Based upon her complaints, what type of headaches is this patient most likely experiencing? Tension headaches 96 | P a g e Who manages healthy people 2020? The United States Department of Health and Human Services Which of the following cranial nerves does not control eye movement? CN III CN II CN VI CN IV CN II A 75-year-old diabetic male requires treatment for pneumonia. Which of the following drugs is not recommended for this patient? Gemifloxacin Moxifloxacin A respiratory fluoroquinolone A macrolide antibiotic A macrolide antibiotic During an office visit, Alex, a 27-year-old male, reports an adverse reaction to mesalamine suppositories and requests an alternative treatment. For which of the following conditions is Alex being treated? Ulcerative colitis hen Maria was 49 years old, she "had a scare" when she discovered a lump in her breast. It was eventually determined to be benign, but, since then, she has been diligent about monitoring herself for other lumps. She has also made sure to receive mammograms annually. Now, at 65, she is asking when these examinations will end. What should you tell her? Ten years from now A human resources (HR) representative has contacted you regarding your 55-year-old patient, Carmen, who has gone through extensive treatment for alcoholism and is now recuperating. The HR representative says that, since the company she works for has paid for her treatment through its health insurance policy, it would like to know whether she has been receptive to the treatments. How do you best respond to the company representative from the choices below? you say that any information regarding Carmen would need written approval from her. You have been treating Doug for asthma. You first prescribed a short-acting B2 adrenergic agonist. After a month or so, he reported that there had been little improvement, so you added an inhaled corticosteroid to the regimen. Now, after another month, you note that Doug's asthma does not present 97 | P a g e with secretions, but he states that his symptoms still have not markedly improved. Of the following, which should you prescribe next? Formoterolong-acting B2 adrenergic agonist Joan, a 46-year-old woman, comes to your office with worries pertaining to a lump she found in her breast. During examination, it is clear she has a mass, although the type is unclear. What is the best diagnostic test to use at this time to distinguish a liquid-filled cyst from a solid mass in the breast? Ultrasound Jordan, 24, presents with complaints of painful, itchy lesions around her labia majora region. She adds, "I think I've had the chills a few times, and I've really felt sluggish for the past few days." Upon physical examination of the affected area, you note small, painful, fluid-filled blisters. Given her presentation and your assessment, which of the following tests should be ordered to confirm the most likely sexually transmitted disease? Viral culture During his check-up, Gary, a 66-year-old male, reports thin stools and recurring bouts of both constipation and diarrhea. He has also experienced rapid, unexplained weight loss in the last month (around 5% of his body weight). Gary is concerned because he has a family history of cancer. Of the following, which procedure would you order to best rule out colon cancer? Colonoscopy A 23-year old male presents with inflammation and itching of the penile head. He reports being in a long-term, monogamous sexual relationship and states that he knows that he and his partner have been faithful for their entire relationship. Which of the following is the most likely diagnosis and treatment? Candida balanitis; topical antifungal Although a chest X-ray is not necessary during an asthma attack, the nurse practitioner would expect to see which of the following on the film if one were taken? = Moderate hyperinflation A 54-year-old former professional athlete is being screened for hypertension, a condition which runs in his family. Which of the following findings is not consistent with a diagnosis of significant hypertension? Epistaxis A S4 heart sound A subparietal headache Blurred vision A subparietal headache